Difference between revisions of "2008 AIME II Problems/Problem 10"

(Solution)
 
Line 49: Line 49:
 
dot((0,0)^^(3,3)^^(1,0)^^(2,3)^^(2,0)^^(0,2)^^(2,1)^^(0,1)^^(1,2),s); draw((0,0)--(3,3)--(1,0)--(2,3)--(2,0)--(0,2)--(2,1)--(0,1)--(1,2));
 
dot((0,0)^^(3,3)^^(1,0)^^(2,3)^^(2,0)^^(0,2)^^(2,1)^^(0,1)^^(1,2),s); draw((0,0)--(3,3)--(1,0)--(2,3)--(2,0)--(0,2)--(2,1)--(0,1)--(1,2));
 
</asy></center>
 
</asy></center>
From <math>(1,2)</math>, there are <math>3</math> possible lengths of <math>\sqrt{1}</math> (to either <math>(1,1),(2,2),(1,3)</math>). Thus, the number of paths is <math>r = 4 \cdot 2 \cdot 3 = 24</math>, and the answer is <math>mr = 10 \cdot 24 = \boxed{240}</math>.  
+
From <math>(1,2)</math>, there are <math>3</math> possible lengths of <math>\sqrt{1}</math> (to either <math>(1,1),(2,2),(1,3)</math>). Thus, the number of paths is <math>r = 4 \cdot 2 \cdot 3 = 24</math>, and the answer is <math>mr = 10 \cdot 24 = \boxed{240}</math>.
  
 
== See also ==
 
== See also ==

Latest revision as of 09:44, 25 November 2019

Problem

The diagram below shows a $4\times4$ rectangular array of points, each of which is $1$ unit away from its nearest neighbors.

[asy] unitsize(0.25inch); defaultpen(linewidth(0.7));  int i, j; for(i = 0; i < 4; ++i) 	for(j = 0; j < 4; ++j) 		dot(((real)i, (real)j)); [/asy]

Define a growing path to be a sequence of distinct points of the array with the property that the distance between consecutive points of the sequence is strictly increasing. Let $m$ be the maximum possible number of points in a growing path, and let $r$ be the number of growing paths consisting of exactly $m$ points. Find $mr$.

Solution

We label our points using coordinates $0 \le x,y \le 3$, with the bottom-left point being $(0,0)$. By the Pythagorean Theorem, the distance between two points is $\sqrt{d_x^2 + d_y^2}$ where $0 \le d_x, d_y \le 3$; these yield the possible distances (in decreasing order) \[\sqrt{18},\ \sqrt{13},\ \sqrt{10},\ \sqrt{9},\ \sqrt{8},\ \sqrt{5},\ \sqrt{4},\ \sqrt{2},\ \sqrt{1}\] As these define $9$ lengths, so the maximum value of $m$ is $10$. For now, we assume that $m = 10$ is achievable. Because it is difficult to immediately impose restrictions on a path with increasing distances, we consider the paths in shrinking fashion. Note that the shrinking paths and growing paths are equivalent, but there are restrictions upon the locations of the first edges of the former.

The $\sqrt{18}$ length is only possible for one of the long diagonals, so our path must start with one of the $4$ corners of the grid. Without loss of generality (since the grid is rotationally symmetric), we let the vertex be $(0,0)$ and the endpoint be $(3,3)$.

[asy] unitsize(0.25inch); defaultpen(linewidth(0.7)); dotfactor = 4; pen s = linewidth(4);  int i, j; for(i = 0; i < 4; ++i) 	for(j = 0; j < 4; ++j) 		dot(((real)i, (real)j)); dot((0,0)^^(3,3),s); draw((0,0)--(3,3));  [/asy]

The $\sqrt{13}$ length can now only go to $2$ points; due to reflectional symmetry about the main diagonal, we may WLOG let the next endpoint be $(1,0)$.

[asy] unitsize(0.25inch); defaultpen(linewidth(0.7)); dotfactor = 4; pen s = linewidth(4); pen c = rgb(0.5,0.5,0.5); int i, j; for(i = 0; i < 4; ++i) 	for(j = 0; j < 4; ++j) 		dot(((real)i, (real)j)); dot((0,0)^^(3,3)^^(1,0),s); draw((0,0)--(3,3),c); draw((3,3)--(1,0));  [/asy]

From $(1,0)$, there are two possible ways to move $\sqrt{10}$ away, either to $(0,3)$ or $(2,3)$. However, from $(0,3)$, there is no way to move $\sqrt{9}$ away, so we discard it as a possibility.

From $(2,3)$, the lengths of $\sqrt{8},\ \sqrt{5},\ \sqrt{4},\ \sqrt{2}$ fortunately are all determined, with the endpoint sequence being $(2,3)-(2,0)-(0,2)-(2,1)-(0,1)-(1,2)$.

[asy] unitsize(0.25inch); defaultpen(linewidth(0.7)); dotfactor = 4; pen s = linewidth(4); pen c = rgb(0.5,0.5,0.5); int i, j; for(i = 0; i < 4; ++i) 	for(j = 0; j < 4; ++j) 		dot(((real)i, (real)j)); dot((0,0)^^(3,3)^^(1,0)^^(2,3)^^(2,0)^^(0,2)^^(2,1)^^(0,1)^^(1,2),s); draw((0,0)--(3,3)--(1,0)--(2,3)--(2,0)--(0,2)--(2,1)--(0,1)--(1,2)); [/asy]

From $(1,2)$, there are $3$ possible lengths of $\sqrt{1}$ (to either $(1,1),(2,2),(1,3)$). Thus, the number of paths is $r = 4 \cdot 2 \cdot 3 = 24$, and the answer is $mr = 10 \cdot 24 = \boxed{240}$.

See also

2008 AIME II (ProblemsAnswer KeyResources)
Preceded by
Problem 9
Followed by
Problem 11
1 2 3 4 5 6 7 8 9 10 11 12 13 14 15
All AIME Problems and Solutions

The problems on this page are copyrighted by the Mathematical Association of America's American Mathematics Competitions. AMC logo.png